7
$\begingroup$

I'm starting to become more functional in Mathcad, so I wanted to take the opportunity to look into known drawbacks, mathematically speaking, of CAS/computer math systems. For instance, one of the exercises we did in school involved "breaking" TI Derive by asking for the limit of a discontinuous function (at the point of discontinuity, of course). Another example would be p. 13 of this;

http://people.virginia.edu/~jcb6t/Mathcad/Johns_Tutorial_on_Everyday_Mathcad.pdf

So basically, I'm hoping folks will contribute known problems with these kinds of apps, whether from their experience with a particular package or generalized problems inherent in computer algebra.

I myself am most concerned with Mathcad and Mathematica, but it would be awesome for people to talk about CAS/computer math in general and drawbacks of various systems (matlab et al) in particular.

  • 0
    Mathoverflow: http://mathoverflow.net/questions/11517/2012-04-30

2 Answers 2

8

There are many other problems: branch cuts, zero-recognition, keeping track of domains of validity, etc. A good place to find such information is to browse the web pages of leading researchers, and conference proceedings (ISSAC,SYMSAC,Sigsam,Eurosam, etc). For example, see Richard Fateman's papers, e.g. his 33 page critique of Mathematica, and Why Computer Algebra Systems Can't Solve Simple Equations and Branch Cuts in Computer Algebra, etc.

  • 3
    My personal impression? Wolfram has a lot in common with the late unlamented Mr. Jobs - his ability to speak marketese overshadows his deficiencies as a scientist/engineer.2012-02-26
3

I only tried this on Mathematica 4, so I don't know if the same problem happens in more modern versions. If you ask Mathematica about $ \int_0^\infty (1-e^{-x})^2/x^2 dx, $ it tells you that the integral does not converge. But if you split the integral, it gives you

$ \int_0^1 (1-e^{-x})^2/x^2 dx\simeq 0.645751, $

$ \int_1^\infty (1-e^{-x})^2/x^2 dx\simeq 0.740543 $

  • 0
    Mathematica 8.0 returns `Log[4]`.2012-02-27